You are on page 1of 4

Real Analysis HW 3 Solutions

Problem 31. Justify the assertion in the proof of Vitalis Theorem that it suffices to
consider the case that E is bounded.
Solution. We know by a previous problem that any positive measure set contains a bounded
set which is also positive measure. Therefore if we can prove Vitalis theorem for any bounded
set of positive measure, we have also proven it for any set of positive measure.

Problem 32. Does Lemma 16 remain true if is allowed to be finite or to be uncountably
infinite? Does it remain true if is allowed to be unbounded.
Solution. Recall Lemma 16 from the text:
Lemma 16 Let E be a bounded measurable set of real numbers. Suppose there
is a bounded, countably infinite set of real numbers for which the collection of
translates of E, ( + E) , is disjoint. Then m(E) = 0.
Lemma 16 does not remain true if is allowed to be finite. Let E = [0, 1) and = {0, 1}.
Then E + 1 and E are disjoint, but E is not measure 0.
If is uncountable, then we may always pick a countable subset which satisfies the conditions
of the theorem. Hence the Lemma holds if is uncountable.
The Lemma is not true if is unbounded. Consider E = [0, 1) and = Z. Then E + are
all disjoint for all .

Problem 33. Let E be a non-measurable set of finite outer measure. Show that there is a
G set G that contains E for which
m (E) = m (G), while m (G E) > 0
T

Proof. Define G in the usual way, G =


k=1 Ok where {Ok }k=1 are open sets satisfying
1

m (Ok ) < m (E) + k . Using monotonicity and that fact that m (G) m (E) + 1/k for
every k 1, we readily conclude that m (G) = m (E). However we cannot have that
m (G E) = 0, since that would imply that E = G (G E)C is measurable. Therefore
m (G E) > 0.

Problem 38. Let the function f : [a, b] R be Lipschitz, that is, there is a constant c 0
such that for all [u, v] [a, b], |f (u) f (v)| c|u v|. Show that f maps a set of measure
zero onto an set of measure zero. Show that f maps an F set onto an F set. Conclude
that f maps a measurable set to a measurable set.
1

Proof: Let  > 0 and let E [a, b] be a set of measure zero, then
Pwe know that there

exists a collection of open intervals {Ik }k=1 that cover E such that k=1 `(Ik ) /c, where
c is the Lipschitz constant for f . Now let ak = inf xIk f (x) and bk = supxIk f (x). Note
that f (Ik ) [ak , bk ]. By the Lipschitz property of f , we know that for any u, v [a, b],
|f (u) f (v)| c |u v|. Taking the sup over u, v Ik on both sides of the inequality, we
find
bk ak = sup |f (u) f (v)| c sup |u v| = c `(Ik ).
u,vIk

u,vIk

It follows that m(f (Ik )) bk ak c `(Ik ), and since f (E)


m(f (E))

m(f (Ik ))

k=1

k=1

f (Ik ) this implies

c `(Ik ) < .

k=1

Therefore f (E) is measure 0.


To show that f maps F sets onto F sets, let F be a F set. By definition
we can write
S
,
F
=
F
F as the countable union of a collection of closed sets {Fk }
k=1
k=1 k . Since f is
continuous, and by assumption Fk are closed and S
bounded and thereby compact, we know
that f (Fk ) is a closed set and therefore the union
k=1 f (Fk ) is an F set. It follows that
!

[
[
f (F ) = f
Fk =
f (Fk )
k=1

k=1

and therefore f maps F sets to F sets.


Finally suppose E [a, b] is a measurable set of positive measure (we already proved the
case for 0 measure), then we know there exists an F set F E such that m(E F ) = 0.
We use the set F to partition E into an F set and a set of measure 0, E = F (E F ).
Therefore
f (E) = f (F ) f (E F ).
However, we know f (F ) is an F set and f (E F ) is a set of measure zero, therefore f (E)
is measurable.

Problem 45. (Not assigned) Show that a strictly increasing function that is defined on
an interval has a continuous inverse.
Solution: Clearly since f : (a, b) Range(f ) is strictly monotone it is one-to-one and so
it has a well defined inverse function f 1 : Range(f ) (a, b). Now fix x Range(f ), and
define the function
fx : 7 f (f 1 (x) + ).
Note fx (0) = x and that since fx () is strictly increasing in , it is invertible with inverse
fx1 (y) = f 1 (y) f 1 (x). Let  > 0, and define
=

min

{ : ||=}

|x fx ()|.

Note that for any y Range(f ) such that |x y| < , there exist 0 , with y = fx (0 ).
Since |x fx ()| is strictly increasing in || we must have |0 | < . This implies that
0 = fx1 (y) = f 1 (y) f 1 (x) and so
|f 1 (y) f 1 (x)| = |0 | < .

Problem 46. Let f be a continuous function and B be a Borel set. Show that f
Borel set.

(B) is a

Proof. Let B denote the Borel sets. We define the set


E = {E R|f 1 (E) B}
to be the collection of sets E for which f 1 (E) is Borel. Note that, since f is continuous
E contains all the open sets. Whats left is to show that E is a -algebra. To do this, we
consider a set E E. We note that f 1 (E C ) = f 1 (E)C B, therefore E C E. Similarly
for any collection of open sets {Ek }
k=1 E we have
!

[
[
f 1
Ek =
f 1 (Ek ) B
k=1

k=1

and therefore k=1 Ek E. It follows that E is a -algebra containing the open sets, and so
by the definition of the Borel sets, B E. Therefore by the definition of E if B B E,
then f 1 (B) B.

Problem 47: (Not assigned) Use the preceding two problems to show that a continuous
strictly increasing function that is defined on an interval maps Borel sets to Borel sets.
Solution: Since f is continuous and strictly monotone, it has a continuous inverse g = f 1
(see problem 45). Since g is continuous, we have by problem 46 that if B is Borel, g 1 (B) =
f (B) is Borel.

Problem 4: Suppose f is a real-valued function on R such that f 1 (c) is measurable for
each number c. Is f necessarily measurable?
Solution: No. Let V be a non-measurable subset of (0, 1) and consider the function f (x) =
ex (2V 1), where V denote the idicator function on V . Note that f > 0 on V and f 0
on R V . Since f is one-to-one we also have that f 1 (c) is either empty or a singleton and
therefore measurable. However, by construction we have f 1 ((0, )) = {x : f (x) > 0} = V .
Therefore f is not measurable.

Problem 6. Let f be a function with measurable domain D. Show that f is measurable if
and only if the function g defined on R by g(x) = f (x) for x D and g(x) = 0 for x
/ D is
measurable.
Solution. Let c be a real number. Suppose that g is measurable, then
{x D : f (x) > c} = {x R : g(x) > c} D
3

is measurable, since {x R : g(x) > c} and D are both measurable, therefore f is measurable. In the case that f is measurable, we see that if c 0, then
{x R : g(x) > c} = {x D : f (x) > c}
and if c < 0, then
{x R : g(x) > c} = Dc {x D : f (x) > c}.
Therefore g is measurable since {x D : f (x) > c} and Dc is.

You might also like